aras2213

New Member
ارسال ها
216
لایک ها
228
امتیاز
0
پاسخ : ماراتن نظریه ی اعداد (سطح ممتاز)

درسته ببخشید امتحانا حواس برام نذاشتن . خب اینبار m=1 قرار بدید که دوباره به چندجمله ای ثابت میرسید.
الان اگه بزاریم m=1 فقط نتیجه میگیریم که برای هر عدد طبیعی مثل n ،
دقت کنید که چند جمله ای
توی شروط مساله صدق میکنه که c عددی صحیح و غیر صفره(در بین جواب های غیر ثابت)
 

golsefatan

New Member
ارسال ها
331
لایک ها
264
امتیاز
0
پاسخ : ماراتن نظریه ی اعداد (سطح ممتاز)

الان اگه بزاریم m=1 فقط نتیجه میگیریم که برای هر عدد طبیعی مثل n ،
دقت کنید که چند جمله ای
توی شروط مساله صدق میکنه که c عددی صحیح و غیر صفره(در بین جواب های غیر ثابت)
پس يا بايد اثبات كنيم كه مساله دو جواب داره يا جواب هاي ديگه رو پيدا كنيم...
 

math1998

New Member
ارسال ها
336
لایک ها
224
امتیاز
0
پاسخ : ماراتن نظریه ی اعداد (سطح ممتاز)

الان اگه بزاریم m=1 فقط نتیجه میگیریم که برای هر عدد طبیعی مثل n ،
دقت کنید که چند جمله ای
توی شروط مساله صدق میکنه که c عددی صحیح و غیر صفره(در بین جواب های غیر ثابت)
این جواب دومیو چه جوری بدست اوردید؟
 

aras2213

New Member
ارسال ها
216
لایک ها
228
امتیاز
0
پاسخ : ماراتن نظریه ی اعداد (سطح ممتاز)

پس يا بايد اثبات كنيم كه مساله دو جواب داره يا جواب هاي ديگه رو پيدا كنيم...
اگه حل من درست باشه احتمالا همین دو جواب رو داره

---- دو نوشته به هم متصل شده است ----

این جواب دومیو چه جوری بدست اوردید؟
دقیقا منظورت رو متوجه نمیشم:92:
اگه منظورت اینه که از کجا باید بفهمیم که این یه جوابه معمولا این جور سوال ها یکی از جواب ها شون همینه که گفتم.اگه منظورت اینه که از کجا بفهمیم که این تنها جواب غیر ثابت مساله هست اولش رو میگم بقیش رو یادم نمیاد ولی روش فکر میکنم اگه یادم اومد حتما منویسم جواب کاملش رو(اگه کسی هم حل کرده جواب رو بنویسه ممنون میشم)
لم:برای هر چند جمله ای با ضرایب صحیح که غیر ثابت است بی نهایت عدد اول مثل q وجود دارد که معادله همنهشتی
جواب طبیعی داشته باشد.
حالا اگه جمله آزاد چند جمله ای غیر صفر باشه طبق لم بالا یه عدد اول مثل q وجود داره که
و برای یه n طبیعی
حالا طبق فرض مساله

که از این نتیجه میگیریم:
که با نحوه ی انتخاب q تناقض داره. پس جمله آزاد چند جمله ای صفره
بعد یادمه این کارو می کردیم:
فرض کنید
که
غیر ثابته و عامل x نداره.بعد هم از ایده ای مشابه قبل استفاده می کردیم یعنی بینهایت تا عدد اول مثل p پیدا میکردیم که یه ویژگی داشته باشند(دقیقا این ویژگی رو یادم نمیاد اما یادمه از قضیه دیریکله برای تصاعد های حسابی و این که بی نهایت عدد اول دارند استفاده می کردیم) که از اون ویژگی بشه نتیچه گرفت:
که چون بینهایت تا عدد اول به این شکل داشتیم نتیجه میشه
هم عامل x داره و این هم تناقضه.
 

math

New Member
ارسال ها
1,129
لایک ها
1,096
امتیاز
0
پاسخ : ماراتن نظریه ی اعداد (سطح ممتاز)

دوستان کسی که سوال رو حل میکنه باید سوال بعدی رو هم بزاره !!!

سوال بعد : تمام چند جمله ای ها با ضرایب صحیح را بیابید که برای هر
که
مربع کامل باشد انگاه
مربع کامل است .
 

nima-1376

New Member
ارسال ها
63
لایک ها
53
امتیاز
0

aras2213

New Member
ارسال ها
216
لایک ها
228
امتیاز
0
پاسخ : ماراتن نظریه ی اعداد (سطح ممتاز)

چشم!
گفتم چون سوال رو خودم گذاشتم و جواب هم همین طور(البته جواب اون سوال هنوز کامل نیست) شاید بهتر باشه سوال بعد رو یکی دیگه بزاره.سوال بعد(سوال قبل رو nima1376 جواب داده اند )
همه توابع f از N به N رو پیدا کنید که برای هر m,n طبیعی
.
کلا همه سوال ها ی چند جمله ای با ضرایب صحیح و تابع های از N به N سوال های قشنگی میشن:87:

---- دو نوشته به هم متصل شده است ----
 
آخرین ویرایش توسط مدیر

math1998

New Member
ارسال ها
336
لایک ها
224
امتیاز
0
پاسخ : ماراتن نظریه ی اعداد (سطح ممتاز)

چشم!
گفتم چون سوال رو خودم گذاشتم و جواب هم همین طور(البته جواب اون سوال هنوز کامل نیست) شاید بهتر باشه سوال بعد رو یکی دیگه بزاره.سوال بعد(سوال قبل رو nima1376 جواب داده اند )
همه توابع f از N به N رو پیدا کنید که برای هر m,n طبیعی
.
کلا همه سوال ها ی چند جمله ای با ضرایب صحیح و تابع های از N به N سوال های قشنگی میشن:87:

---- دو نوشته به هم متصل شده است ----
f(i),i=1,2,3 را حساب کنید بعد m=3,n=4 قرار دهید که به تناقض میرسه.
 

math1998

New Member
ارسال ها
336
لایک ها
224
امتیاز
0

golsefatan

New Member
ارسال ها
331
لایک ها
264
امتیاز
0
پاسخ : ماراتن نظریه ی اعداد (سطح ممتاز)

چشم!
گفتم چون سوال رو خودم گذاشتم و جواب هم همین طور(البته جواب اون سوال هنوز کامل نیست) شاید بهتر باشه سوال بعد رو یکی دیگه بزاره.سوال بعد(سوال قبل رو nima1376 جواب داده اند )
همه توابع f از N به N رو پیدا کنید که برای هر m,n طبیعی
.
کلا همه سوال ها ی چند جمله ای با ضرایب صحیح و تابع های از N به N سوال های قشنگی میشن:87:

---- دو نوشته به هم متصل شده است ----
من اثبات کردم
f(n) از n کمتر-مساوی است. کافی است اثبات این که f(n) از n بیشتر-مساوی است.
 

aras2213

New Member
ارسال ها
216
لایک ها
228
امتیاز
0
پاسخ : ماراتن نظریه ی اعداد (سطح ممتاز)

من اثبات کردم
f(n) از n کمتر-مساوی است. کافی است اثبات این که f(n) از n بیشتر-مساوی است.
بله.من همچین راهی رو براش ندیدم ولی خب شاید این جوری هم حل شه
سوال بعد:
آیا دو دنباله اکیدا صعودی
از اعداد طبیعی وجود دارند که برای هر عدد طبیعی n، داشته باشیم
؟
لطفا اگه کسی حل کرد سوال بعدی رو بزاره:196:
 

golsefatan

New Member
ارسال ها
331
لایک ها
264
امتیاز
0
پاسخ : ماراتن نظریه ی اعداد (سطح ممتاز)

بله.من همچین راهی رو براش ندیدم ولی خب شاید این جوری هم حل شه
سوال بعد:
آیا دو دنباله اکیدا صعودی
از اعداد طبیعی وجود دارند که برای هر عدد طبیعی n، داشته باشیم
؟
لطفا اگه کسی حل کرد سوال بعدی رو بزاره:196:
ممنون از سوال های خوبی که می گذارید ولی خوبه بذارید سوال حل بشه بعد سوال بذارید. وقتی سوال کامل حل شد راه حل قرار داده میشه و همون نفر سوال بعدی رو میذاره.
 

aras2213

New Member
ارسال ها
216
لایک ها
228
امتیاز
0
پاسخ : ماراتن نظریه ی اعداد (سطح ممتاز)

ممنون از سوال های خوبی که می گذارید ولی خوبه بذارید سوال حل بشه بعد سوال بذارید. وقتی سوال کامل حل شد راه حل قرار داده میشه و همون نفر سوال بعدی رو میذاره.
چشم!
ولی آقای nima1376 سوال قبل رو لینکش رو گذاشتند ولی سوال جدید نذاشتند:2:
من هم این سوال رو گذاشتم که دوباره تاپیک نخوابه
راه حل سوال قبل رو توی این لینک میتونید مشاهده کنید:
AoPS Forum - [f^2(m)+f(n)]|(m^2+n)^2 • Art of Problem Solving
بازم ممنون:53:
 

nima-1376

New Member
ارسال ها
63
لایک ها
53
امتیاز
0
پاسخ : ماراتن نظریه ی اعداد (سطح ممتاز)

بله.من همچین راهی رو براش ندیدم ولی خب شاید این جوری هم حل شه
سوال بعد:
آیا دو دنباله اکیدا صعودی
از اعداد طبیعی وجود دارند که برای هر عدد طبیعی n، داشته باشیم
؟
لطفا اگه کسی حل کرد سوال بعدی رو بزاره:196:
فرض میکنیم $n$ عددی باشید که همه عواملش به شکل
باشه.
اما

خب به راحتی با باقی مانده چینی میفهمیم چنین دنباله ای وجود دارد.




سوال بعد
iran tst 2014 problem 3 day2
ثابت کنید برای هر k>1
تعداد جواب های طبیعی معادله زیر متناهی است.
 

aras2213

New Member
ارسال ها
216
لایک ها
228
امتیاز
0
پاسخ : ماراتن نظریه ی اعداد (سطح ممتاز)

فرض میکنیم $n$ عددی باشید که همه عواملش به شکل
باشه.
اما

خب به راحتی با باقی مانده چینی میفهمیم چنین دنباله ای وجود دارد.




سوال بعد
iran tst 2014 problem 3 day2
ثابت کنید برای هر k>1
تعداد جواب های طبیعی معادله زیر متناهی است.
خواستم جواب رو بنويسم بعد ديدم كه ترجمه ى همون چيزى ميشه كه تو mathlinks هه!براى همين با اين كه كلا از لينك دادن خوشم نمياد اينم لينك سوالAoPS Forum - question 3 • Art of Problem Solving
كه نميدونم چرا هر چى تلاش ميكنم نميتونم اون جورى بزارمش كه آبى شه و بشه روش كليك كرد!خودتون زحمتش رو بكشيد ديگه!
براى دوستانى هم كه نميخواند جواب رو به صورت كامل ببينند:
به اون معادله به شكل يه چند جمله اى نگاه كنيد و از اين لم استفاده كنيد:اگر يه چندجمله اى با ضرايب صحيح به ازاى بى نهايت عدد طبيعى مربع(توان n ام كامل شه) اون وقت مربع(توان n ام)يك چندجمله اى ديگر با ضريب گويااست.(تصحيح شد)

---- دو نوشته به هم متصل شده است ----

خواستم جواب رو بنويسم بعد ديدم كه ترجمه ى همون چيزى ميشه كه تو mathlinks هه!براى همين با اين كه كلا از لينك دادن خوشم نمياد اينم لينك سوالAoPS Forum - question 3 • Art of Problem Solving
كه نميدونم چرا هر چى تلاش ميكنم نميتونم اون جورى بزارمش كه آبى شه و بشه روش كليك كرد!خودتون زحمتش رو بكشيد ديگه!
براى دوستانى هم كه نميخواند جواب رو به صورت كامل ببينند:
به اون معادله به شكل يه چند جمله اى نگاه كنيد و از اين لم استفاده كنيد:اگر يه چندجمله اى با ضرايب صحيح به ازاى بى نهايت عدد طبيعى مربع(توان n ام كامل شه) اون وقت مربع(توان n ام)يك چندجمله اى ديگر با ضريب گويا است(تصحيح شد)
مثل اين كه خود به خود آبى ميشه!
 
آخرین ویرایش توسط مدیر

nima-1376

New Member
ارسال ها
63
لایک ها
53
امتیاز
0
پاسخ : ماراتن نظریه ی اعداد (سطح ممتاز)

به اون معادله به شكل يه چند جمله اى نگاه كنيد و از اين لم استفاده كنيد:اگر يه چندجمله اى با ضرايب صحيح به ازاى بى نهايت عدد طبيعى مربع(توان n ام كامل شه) اون وقت مربع(توان n ام)يك چندجمله اى ديگر با ضريب صحيح است.
این لم غلطه
اون وقت مربع(توان n ام)يك چندجمله اى ديگر با ضريب گویا است.

در ضمن تو لینک راه حل کاملی وجود ندارد.
 

aras2213

New Member
ارسال ها
216
لایک ها
228
امتیاز
0
پاسخ : ماراتن نظریه ی اعداد (سطح ممتاز)

این لم غلطه
اون وقت مربع(توان n ام)يك چندجمله اى ديگر با ضريب گویا است.

در ضمن تو لینک راه حل کاملی وجود ندارد.
بله.الان درستش ميكنم
چرا وجود داره http://www.renyi.hu/~p_erdos/1975-46.pdf كه تعميم اين مساله هستش(طبق گفته ى اون كسى كه اين رو گذاشته!)
البته من هم يه كليتى از راه حل ml رو نگاه كردم و فكر كردم حلم درسته.يعنى گفتم بقيش آسونه و ....:65:!ولى الان ميبينم كه يه اشتباه كوچولو(!) كردم و بايد يكم بيشتر فكر كنم ولى فكر ميكنم اون چيزى كه توى pdf هه هستش كامله البته يه رو خونى كردمو فقط حكم مساله رو توش ديدم و متوجه شدم كه درباره ى اون داره صحبت ميكنه!
بيشتر روش فكر ميكنم و اگه حل شد جواب رو مينويسم
ممنون
 
آخرین ویرایش توسط مدیر

nima-1376

New Member
ارسال ها
63
لایک ها
53
امتیاز
0
پاسخ : ماراتن نظریه ی اعداد (سطح ممتاز)

بله.الان درستش ميكنم
چرا وجود داره http://www.renyi.hu/~p_erdos/1975-46.pdf كه تعميم اين مساله هستش(طبق گفته ى اون كسى كه اين رو گذاشته!)
فک میکنم این pdf سطح دانشگاهی داره نه المپیادی.
این سوال راه حل المپیادی داره ...
خب من راه حل کامل رو در mathklink مینویسم تا همه استفاده کنند
 
آخرین ویرایش توسط مدیر

aras2213

New Member
ارسال ها
216
لایک ها
228
امتیاز
0
پاسخ : ماراتن نظریه ی اعداد (سطح ممتاز)

فک میکنم این pdf سطح دانشگاهی داره نه المپیادی.
این سوال راه حل المپیادی داره ...
خب من راه حل کامل رو در mathklink مینویسم تا همه استفاده کنند
فكر كنم اگه جمله آزاد رو تو دو طرف مقايسه كنيم حل شه

---- دو نوشته به هم متصل شده است ----

يعنى بدست مياد !(2n) مربع كامل هستش كه اين هم درست نيست چون كافيه بزرگترين عامل اول كوچيكتر از ٢n رو بگيريم
صد در صد جوب زدم چون نبايد اين قدر آسون باشه!

---- دو نوشته به هم متصل شده است ----

2n همون k هه
 
بالا